Đến nội dung

Bui Ba Anh nội dung

Có 530 mục bởi Bui Ba Anh (Tìm giới hạn từ 20-04-2020)



Sắp theo                Sắp xếp  

#664333 Chứng minh tồn tại hình chữ nhật có các đỉnh cùng màu.

Đã gửi bởi Bui Ba Anh on 10-12-2016 - 23:04 trong Tổ hợp và rời rạc

Trong mặt phẳng tọa độ $Oxy$, xét các điểm $(a;b)$ thỏa $a,b$ đều nguyên và $1 \leq a \leq 12$ và $1 \leq b \leq 10$. Các điểm này được tô bởi một trong ba màu khác nhau. Chứng minh rằng tồn tại một hình chữ nhật có các đỉnh là các đỉnh đang xét, các cạnh song song với các trục tọa độ và các đỉnh tô cùng màu.




#648666 $x+2y+3z=100$ có bao nhiêu nghiệm nguyên không âm

Đã gửi bởi Bui Ba Anh on 08-08-2016 - 23:04 trong Số học

1. Phương trình $x+2y+3z=100$ có bao nhiêu nghiệm nguyên không âm?

2. Chứng minh trường hợp tổng quát $(100->n)$




#644679 Thảo luận về Đề thi và Lời giải của IMO 2016

Đã gửi bởi Bui Ba Anh on 12-07-2016 - 16:12 trong Thi HSG Quốc gia và Quốc tế

Bài 4: Ta gọi tập thỏa mãn đề bài là tập chuẩn

Giả sử với số $b$ nào đó ta thu được tập chuẩn $X$, ta sẽ tìm giá trị nhỏ nhất của $b$

Với mỗi phần tử trong  $X$, nếu nó có ước nguyên tố chung với phần tử nào đó trong các phần tử còn lại thì ta xếp các ước nguyên tố đó vào tập $P$. Sau quá trình này, nếu có nhiều phần tử trùng nhau trong $P$ thì ta chọn một trong chúng

Ta sẽ xét các trường hợp

*TH1: Nếu trong $P$ tồn tại các số nguyên tố lớn hơn hoặc bằng $11$, gọi $p$ là một số như vậy

Khi đó tồn tại $P(a+m)$ và $P(a+n)$ chia hết cho $p$ ($m>n$)

Suy ra $$p|P(a+m)-P(a+n)$$

$$=>p|(m-n)(m+n+1)$$

-Nếu $p|m-n$, khi đó do $m$ khác $n$ nên $m \geq 12$ hay $b \geq 12$

-Nếu $p|m+n+1$ khi đó do $m$ khác $n$ nên $m \geq 6$ hay $ b\geq 6$

Vậy trong trường hợp này $b \geq 6$

*TH2: Nếu trong $P$ chỉ chứa các số nguyên tố $3,5,7$ (vì các phân tử đều là số lẻ)

Rõ ràng $P$ không chưa $5$ vì $n^2+n+1$ không chia hết cho $5$

Ta có $2$ nhận xét:

1. $n^2+n+1$ chia hết cho $3$ khi và chỉ khi $n=1$ mod 3, cũng suy ra là trong ba số liên tiếp thì chỉ có một số $t$ mà $P(t)$ chia hết cho $3$

2.$n^2+n+1$ chia hết cho $7$ khi và chỉ khi $n=2,4$ mod 7, cũng suy ra là trong ba số liên tiếp thì có nhiều nhất hai số mà $P$ của nó chia hết cho $7$,trong 2 số liên tiếp thì có nhiều nhất một số mà $P$ của nó chia hết cho $7$

Trở lại trường hợp này

-Nếu $X$ có $2$ phần tử, điều này vô lí vì số phần tử chia hết cho $3$ là $0$ hoặc $2$, nếu là $0$ thì hai số đó đều chia hết cho $7$ vô lí theo nhận xét. nếu có $2$ số chia hết cho $3$ cũng vô lí theo nhận xét

-Nếu $X$ có $3$ phần tử, nếu có $0$ phần tử chia hết cho $3$, suy ra tất cả đều chia hết cho $7$ là vô lí theo nx2. Nếu có $2,3$ số chia hết cho $3$ vô lí theo nx1.

-Nếu $X$ có $4$ phần tử, nếu có $0$ phần tử chia hết cho $3$ là vô lí theo nx1, nếu có nhiều hơn $2$ phần tử chia hết cho $3$ thì chỉ có thể là $P(a+1),P(a+4)$, suy ra $P(a+2),P(a+3)$ chia hết cho $7$ vô lí theo nx2

-Nếu có $5$ phần tử: Có $0$ phần tử chia hết cho $3$ là vô lí, nếu có $2$ phần tử chia hết cho $3$, dễ thấy có $2$ phần tử liên tiếp trong $X$ chia hết cho $7$ vô lí theo nx2.

Tóm lại trong trường hợp này $b \geq 6$

Ta sẽ chỉ ra min $b$ là 6

Xét số $n$ thỏa mãn: n=0 mod 3, n=5 mod 11, n=6 mod 7. Số này là tồn tại theo định lí số dư trung hoa

Khi đó $3 \mid P(x+1), P(x+4),19 \mid P(x+2),P(x+6),  7 \mid P(x+3), 7 \mid P(x+5)$.




#644558 Tìm giá trị nhỏ nhất có thể của $n$?

Đã gửi bởi Bui Ba Anh on 11-07-2016 - 20:15 trong Tổ hợp và rời rạc

Cho bảng $8\times 6$,các ô của bảng được tô bởi $n$ màu sao cho mỗi cặp 2 màu chỉ xuất hiện cùng nhau không quá một hàng. Tìm giá trị nhỏ nhất có thể của $n$?

Đề bài khá lỏng lẻo vì nếu vậy thì ta cứ tô các ô hết một màu, đến hàng cuối cùng tô một màu nữa là xong nếu $n>1$

Nếu thêm điều kiện không có $2$ hàng nào mà tất cả các ô chung một màu:xét bài toán tổng quát là tô màu cho bảng $m.x$

Thay việc tô màu bằng việc đánh các số vào ô vuông

Giả sử ta dùng $k$ số từ $1$ đến $k$ để điền vào ô vuông thỏa mãn đề bài

Với $k$ hàng đầu tiên, với hàng $i$ ta đánh hết các số trong hàng là $i$.

Còn lại $m-k$ hàng

Với mỗi hàng như vậy, ta chọn một cặp bất kì $2$ số từ $k$ số đầu điền vào, các số còn lại trong hàng, ta đánh hết chúng là một trong hai số từ cặp đã chọn.

Như vậy để thuật toán này đúng thì $C^2_k \geq m-k$ hay $k \geq \dfrac{-1+\sqrt{1+8m}}{2}$ với k nguyên

Phần lập luận chứng minh k nhỏ nhất tương tự như lập luận tìm k này




#644508 Thảo luận về Đề thi và Lời giải của IMO 2016

Đã gửi bởi Bui Ba Anh on 11-07-2016 - 15:34 trong Thi HSG Quốc gia và Quốc tế

Bài $1$ khá hay. Gọi $CD$ cắt $BF$ tại $T$ và dễ dàng chứng minh $E$ là trung điểm $FT$ (Chú ý cách xác định điểm $E$ là giao của trung trực $AD$ và đường thẳng đổi xứng của $AC$ qua $AD$)

Khi đó tứ giác $XMFE$ nội tiếp, ngũ giác $BMDEA$ và $BFDXC$ nội tiếp.

Suy ra $EM,FX,BD$ đồng quy tại tâm đẳng phương của ba đường tròn.




#636602 VMF's Marathon Hình học Olympic

Đã gửi bởi Bui Ba Anh on 29-05-2016 - 19:48 trong Hình học

Mình xin lỗi vì quên đăng bài đề nghị

$\boxed{\text{Bài toán 15.}}$  Đường tròn $W_1$ và $W_2$ giao nhau tại $P,K$. $XY$ là tiếp tuyến chung ngoài gần $P$ hơn của $W_1,W_2$ với $X$ thuộc $W_1$ và $Y$ thuộc $W_2$. $XP$ cắt $W_2$ tại điểm thứ hai $C$ và $YP$ cắt $W_1$ tại điểm thứ hai $B$. Gọi $A$ là giao của $BX,CY$. Chứng minh rằng nếu $Q$ là giao điểm còn lại của $(ABC)$ và $(AXY)$ thì góc $QXA$ bằng góc $QKP$

Nguồn: sưu tầm

 

 




#636385 VMF's Marathon Hình học Olympic

Đã gửi bởi Bui Ba Anh on 28-05-2016 - 22:17 trong Hình học

$\boxed{\text{Giải bài 14.}}$ Gọi $D,E,F$ là tiếp điểm của $(I)$ với $BC,AC,AB$, giả sử $B_1,C_2$ thuộc $BC$,$B_2,A_1$ thuộc $AB$, $A_2,C_1$ thuộc $AC$, gọi các đường tròn nhỏ là $(u),(v),(w)$ thứ tự ứng với đỉnh $A,B,C$

Ta sẽ chứng minh $XD,YE,ZF$ đồng quy tại $P$

Thật vậy dễ thấy các tam giác $B_1XC_2,B_2A_1Z,YA_2C_1$ nội tiếp. (1)

Mà $\dfrac{sin(A_1ZF)}{sin(FZB_1)}=\dfrac{A_1F.ZB_2}{B_2F.ZA_1}$, thiết lập các đẳng thức tương tự và chú ý (1), áp dụng Ceva dạng sin ta có đpcm

Mặt khác tam giác $DEF$ đồng dạng $XYZ$ và $XD,YE,ZF$ đồng quy tại $P$ nên tồn tại phép vị tự tâm $P$ biến $DEF$ thành $XYZ$, do đó biến $I$ thành $M$ nên $P,I,M$ thẳng hàng.

Giả sử tiếp tuyến chung trong của $(v)$ và $(I)$ cắt $BD,XD,FZ$ tại $K,L,R$ và tiếp tuyến chung trong của $(w)$ và $(I)$ cắt $CD,XD,EY$ tại $H,L',Q$

Do $B_1K=KD$ và $DH=C_2H$ nên $DL=LX$ và $DL'=L'X$ theo talet hay các tiếp tuyến này cắt nhau tại một điểm trên $DX$

Chứng minh tương tự cho các tiếp tuyến chung còn lại

Khi đó tam giác $LQR$ đồng dạng $XYZ$ và $XL,YQ,CR$ đồng quy tại $P$ nên tồn tại phép vị tự biến $LQR$ thành $XYZ$ và do đó biến $I$ thành $N$ hay $P,I,N$ thẳng hàng

Vậy ta có đpcm.

$$\begin{array}{| l | l |} \hline Ngockhanh99k48 & 1\\ \hline IHateMath & 1\\ \hline fatcat12345 & 2\\ \hline dogsteven & 3\\ \hline baopbc & 4\\ \hline QuangDuong12011998 & 1\\ \hline xuantrandong & 1\\ \hline mrjackass & 1\\ \hline vietnaminmyheart & 1\\ \hline BuiBaAnh & 1\\ \hline\end{array}$$




#636265 Chứng minh $\frac{1}{AB}$$=\frac{1}{BC}+\frac{1...

Đã gửi bởi Bui Ba Anh on 28-05-2016 - 15:48 trong Hình học

xem một bài viết ở :https://hokagealtctr...uyen-sinh-thpt/




#632189 ĐỀ THI OLYMPIC CHUYÊN KHOA HỌC TỰ NHIÊN 2016

Đã gửi bởi Bui Ba Anh on 10-05-2016 - 00:10 trong Thi HSG cấp Tỉnh, Thành phố. Olympic 30-4. Đề thi và kiểm tra đội tuyển các cấp.

anh em thấy cách dồn này sao sao ấy vì anh đặt t=√bc thì cái đk đầu bài đâu có bằng đk dồn biến và ta thấy rõ là 2at khác với ab+ac

Để mình xem lại




#632008 ĐỀ THI OLYMPIC CHUYÊN KHOA HỌC TỰ NHIÊN 2016

Đã gửi bởi Bui Ba Anh on 08-05-2016 - 21:23 trong Thi HSG cấp Tỉnh, Thành phố. Olympic 30-4. Đề thi và kiểm tra đội tuyển các cấp.

 

Câu 7. Cho $a, b, c$ là các số thực dương thỏa mãn $ab + bc + ca + 2abc = 1$. Chứng minh rằng $$\sum\frac{a(a + 1)}{(2a + 1)^{2}}\le \frac{9}{16}$$

 

 

 

Tiếp cận bằng dồn biến

Đặt $f(a,b,c)=\dfrac{a(a+1)}{(2a+1)^2}+\dfrac{b(b+1)}{(2b+1)^2}+\dfrac{c(c+1)}{(2c+1)^2}$

Ta sẽ chứng minh $f(a,t,t) \leq \dfrac{9}{16}$ với $t$ thỏa mãn $2at^2+2at+t^2=1$

Tức là chứng minh $\dfrac{a(a+1)}{(2a+1)^2}+2\dfrac{t(t+1)}{(2t+1)^2} \leq \dfrac{9}{16}$

$$<=> f(t)=\dfrac{1-t^2}{4}+2\dfrac{t(t+1)}{(2t+1)^2} \leq \dfrac{9}{16}$$

Xét $f'(t)=\dfrac{-t}{2}+\dfrac{2}{(2t+1)^3}=0<=>(2t-1)(4t^3+8t^2+7t+4) =0<=>t=0,5$

Dùng bảng biến thiên dễ suy ra $f(t) \leq f(0,5)=\dfrac{9}{16}$

Tiếp theo ta sẽ chứng minh $f(a,b,c) \leq f(a,t,t)$ với $t =\sqrt{bc}(1)$

Khi đó nếu $(1)$ đúng thì ta có thể đưa về trường hợp trên và ta có đpcm

$$f(a,b,c) \leq f(a,t,t)$$

$$<=>\dfrac{b(b+1)}{(2b+1)^2}+\dfrac{c(c+1)}{(2c+1)^2} \leq \dfrac{2\sqrt{bc}(\sqrt{bc}+1)}{(2\sqrt{bc}+1)^2}$$

$$<=>(y-z)^2.\dfrac{-y^3z^3+(y^2+yz+z^2)-4y^2z^2(y^2+yz+z^2)-4yz(y^2+yz+z^2)-3yz+1}{(2y^2+1)^2(2z+1)^2(2yz+1)^2} \leq 0(b=y^2,c=z^2)(2)$$

Đặt $m=y^2+yz+z^2$,$n=yz$,$m \geq 3n$ 

$$(2)<=>m(1-4n^2-4n)-4n^3-2n+1 \leq0$$

Do $2abc+ab+ac+bc=1$ nên không mất tổng quát ta có thể giả sử $bc \geq \dfrac{1}{4}=>n \geq \dfrac{1}{2}=>1-4n^2-4n<0$

Suy ra $m(1-4n^2-4n)-4n^3-2n+1 \leq -16n^3-12n^2+n+1=g(n) \leq 0$ với $1 >n \geq \dfrac{1}{2}$. Điều này đúng do $g(n)$ nghịch biến trên tập xác định




#628386 xác định số $n$ bé nhất sao cho từ các điều kiện cho trước

Đã gửi bởi Bui Ba Anh on 19-04-2016 - 23:27 trong Tổ hợp - Xác suất và thống kê - Số phức

Biểu thị mỗi học sinh là một điểm trên mặt phẳng sao cho không có ba điểm nào thẳng hàng.

Cứ $2$ học sinh đạt điểm tối ưu cho cùng một môn được nối với nhau bởi 1 đoạn thẳng xác định bởi 1 màu nào đó

Do điều kiện của bài toán cứ $3$ học sinh đạt điểm tối ưu cho một môn sẽ biểu thị bằng một tam giác có $3$ cạnh cùng màu, và giữa $2$ tam giác bất kì có đúng $1$ điểm chung

Nhận xét 1: Nếu $4$ tam giác, với mỗi tam giác chỉ có các cạnh cùng màu, có chung $1$ đỉnh, thì tất cả các tam giác khác sẽ cùng có chung đỉnh này.

Thật vậy. Xét $4$ tam giác này, giả sử tam giác thứ $5$ có đỉnh không là đỉnh chung của $4$ tam giác này, do điều kiện $2$ tam giác bất kì có $1$ đỉnh chung nên theo $\Dirichle$ tồn tại $2$ tam giác trong $4$ tam giác đó có chung đỉnh với tam giác thứ $5$. Suy ra $2$ tam giác này có $2$ đỉnh chung, vô lí. Nên điều giả sử sai và ta có đpcm

Nhận xét 2: Điều kiện $n=4$ là điều kiện bé nhất để tất cả các tam giác chung đỉnh.( $n \geq 3$ nên dễ dàng tìm phản ví dụ cho $3$)

Trở lại bài toán

-Với $n$ tam giác, xét một tam giác bất kì, từ nhận xét, suy ra phải tồn tại $1$ đỉnh của nó chung đỉnh của $3$ tam giác khác

-Theo $\Dirichle$ suy ra số tam giác bé nhất để đảm bảo luôn có $3$ tam giác chung đỉnh với tam giác đang xét là:$3.2+1=7$ tam giác. Cộng với tam giác đang xét. Suy ra $n=8$




#627639 $(O),(I),(A)$ có trục đẳng phương chung

Đã gửi bởi Bui Ba Anh on 17-04-2016 - 00:47 trong Hình học

Chứng minh rằng đường tròn nội tiếp $(I)$ của tam giác $ABC$, đường tròn $A$-mixtilinear, và đường tròn tâm $A$ bán kính $AI$ có trục đẳng phương chung




#627532 $\dfrac{KQ}{KB}=\dfrac{TC}{...

Đã gửi bởi Bui Ba Anh on 16-04-2016 - 18:01 trong Hình học

Cho tứ giác nội tiếp $ABCD$, điểm $Miquel$ $M$.$AC$ cắt $BD$ tại $Q$, $AB$ cắt $CD$ tại $P$. Gọi $T,K$ là hai điểm thuộc $PC,QB$. Đường thẳng $TK$ cắt $QC$ và $PB$ lần lượt ở $U,V$. Chứng minh rằng $4$ điểm $A,M,U,V$ đồng viên khi và chỉ khi $\dfrac{KQ}{KB}=\dfrac{TC}{TP}$




#616796 $\left\{\begin{matrix} xy^2-y^2+x-xy+4y=1...

Đã gửi bởi Bui Ba Anh on 25-02-2016 - 00:44 trong Phương trình - hệ phương trình - bất phương trình

Giải hệ phương trình trên tập số thực:

$\left\{\begin{matrix} xy^2-y^2+x-xy+4y=1\\x^3+x^2-xy=2 \end{matrix}\right.$




#609567 $2\sum \dfrac{AA_2}{AA_1} \geq 1+\dfrac{4r}{R}$

Đã gửi bởi Bui Ba Anh on 18-01-2016 - 01:12 trong Hình học

Cho tam giác $ABC$ nội tiếp đường tròn $(O)$ bán kính $R$, ngoại tiếp đường tròn $(I)$ bán kính $r$. Các đường trung tuyến $AA_1,BB_1,CC_1$

Tiếp tuyến tại $B,C$ của đường tròn $(O)$ cắt nhau tại $S$, $AS$ cắt $BC$ tại $A_2$. Các điểm $B_2,C_2$ xác định tương tự

Chứng minh rằng:

$\dfrac{AA_2}{AA_1}+\dfrac{BB_2}{BB_1}+\dfrac{CC_2}{CC_1} \geq \dfrac{1}{2}+\dfrac{2r}{R}$




#609290 Nếu $f(f(x))=0$ thì thay $x$ bởi $y$, ta có...

Đã gửi bởi Bui Ba Anh on 16-01-2016 - 19:57 trong Kinh nghiệm học toán

nó cũng chỉ là cái tên thôi mà em, chữ nào cũng đc, coi như một phép thay (điều kiện $x,y$ cùng trên 1 miền nào đó)




#608021 $a-b|a$ và $a-b|b$

Đã gửi bởi Bui Ba Anh on 08-01-2016 - 20:19 trong Số học

Ta chứng minh bằng qui nạp. Giả sử bài toán đúng tới $n$ tức là tồn tại tập $S_{n}=\left \{ a_1,a_2,...a_{n} \right \}$  thỏa đề

Ta xây dựng tập $S_{n+1}$ như sau : Đặt $A=\prod (a_i-a_j)$ với $i,j$ chạy từ $1$ đến $n$

Với mỗi $k$ từ $1$ đến $n$ ta chọn $b_k=Aa_1a_2...a_n+a_k$ . Còn $b_{k+1}=Aa_1a_2...a_n$

Ta thấy $b_i-b_j= a_i-a_j |a_i|b_i$ và $b_i-b_j= a_i-a_j |a_j|b_j$ ; $\forall 1\leq i,j\leq n$

và $b_i-b_j$ ko là ước của $b_k$ nào nữa

Riêng $b_i-b_{n+1}= a_i |gcd(b_i,b_{n+1});\forall 1\leq i\leq n$ và cũng ko là ước của $b_k$ nào nữa.

Vậy tập $S_{n+1}=\left \{ b_1,b_2,...b_{n+1} \right \}$ thỏa đề và ta có $đpcm$

Dạ anh giải thích rõ hơn chỗ này được không ạ? Vì giả sử là ước của một $b_k$ nào đó thì $a_i|a_k$ Từ chỗ này em vẫn chưa tìm ra vô lí.




#607959 $a-b|a$ và $a-b|b$

Đã gửi bởi Bui Ba Anh on 08-01-2016 - 15:17 trong Số học

Chứng minh rằng với mỗi số tự nhiên $n$ lớn hơn $1$, luôn tồn tại một tập $S$ gồm $n$ phần tử sao cho với hai phần tử $a,b$ bất kì trong $S$, $a-b|a$ và $a-b|b$ nhưng $a-b$ không là ước của bất kì số nào trong $S$ khác nữa.




#605343 $S(n)=n^2-14$

Đã gửi bởi Bui Ba Anh on 26-12-2015 - 17:25 trong Số học

Với mỗi số nguyên dương $n$, ta định nghĩa $S(n)$ là số nguyên dương lớn nhất sao cho với mỗi số nguyên dương $k \leq S(n)$, $n^2$ có thể viết được thành tổng của $k$ số chính phương. Tìm $1$ ví dụ $n$ sao cho $S(n)=n^2-14$ và giải thích nguyên nhân vì sao nghĩ ra con số đó  :D




#603355 Đề cử Thành viên ấn tượng năm 2015 của Diễn đàn Toán học

Đã gửi bởi Bui Ba Anh on 15-12-2015 - 20:15 trong Thông báo tổng quan

1.Tên ứng viên: Zaraki

2.Thành tích-Đóng góp nổi bật: thành viên trong ban đề xướng, ra đề và chấm thi VMEO 2015, dù đã năm 11 và nhiều việc hơn nhưng vẫn thường xuyên tham gia và đóng góp cho diễn đàn. Hoàn toàn xuất phát từ đam mê và tâm huyết với VMF

3. Chú thích:nope




#596821 Từ $M$ có thể lập được bao nhiêu số có $15$ chữ số

Đã gửi bởi Bui Ba Anh on 04-11-2015 - 17:28 trong Tổ hợp và rời rạc

Số các số có thể lập:$\frac{15!}{3!^{5}}$
nhưng trong đó có các số có số $0$ đứng đầu:$\frac{14!}{3!^{4}.2!}$.
Vậy số các số lập được theo ycđb:
$\frac{15!}{3!^{5}}-\frac{14!}{3!^{4}.2!}=\frac{2.14!}{3!^{4}}=134534400$ số

Mình k rõ công thức này lắm




#596741 Từ $M$ có thể lập được bao nhiêu số có $15$ chữ số

Đã gửi bởi Bui Ba Anh on 03-11-2015 - 21:23 trong Tổ hợp và rời rạc

Cho tập $M=(0,1,2,3,4)$. Từ $M$ có thể lập bao nhiêu số có $15$ chữ số sao cho mỗi số trong $M$ xuất hiện đúng $3$ lần




#594818 chọn đội tuyển tỉnh Đaklak 2015-2016

Đã gửi bởi Bui Ba Anh on 22-10-2015 - 15:24 trong Thi HSG cấp Tỉnh, Thành phố. Olympic 30-4. Đề thi và kiểm tra đội tuyển các cấp.

 

KÌ THI LẬP ĐỘI TUYỂN DỰ THI QUỐC GIA -NGÀY 1

Câu 3 (5 điểm)

Cho tam giác $ABC$,$M$ là điểm nằm trong của tam giác.Gọi khoảng cách từ $M$ đến cạnh $AB,BC,CA$ lần lượt là $d_c,d_a,d_b$ và khoảng cách từ $M$ đến các đỉnh $A,B,C$ lần lượt là $x,y,z$.Chứng minh rằng

$\frac{x+y+z}{d_a+d_b+d_c}\ge 2$

 

Câu 4 (5 điểm)

Giải hệ phương trình

$\left\{\begin{matrix} 2(x+y+z)-15=0\\2x+4y+7z-2xyz=0 \\\frac{1}{\sqrt{3x}}+\sqrt{10y}+\sqrt[4]{8z}=\frac{22}{3} \end{matrix}\right.$

 

Chắc câu 1 và 2 là khó của đề 

ý tưởng câu 3 và 4

Câu 3: dễ thấy định lý Erdos-mordel

câu 4: VNTST 2001




#594479 một hộp có 40 thẻ được đánh thứ tự từ 1->40 .Hỏi có bao nhiêu cách lấy 3...

Đã gửi bởi Bui Ba Anh on 19-10-2015 - 20:51 trong Tổ hợp - Xác suất và thống kê - Số phức

một hộp có 40 thẻ được đánh thứ tự từ 1->40 .Hỏi có bao nhiêu cách lấy 3 tấm thẻ , trong đó tổng các số ghi trên 3 tấm thẻ chia hết cho 3

Số trên ba tấm thể nhận số dư lần lượt là $(0,0,0);(1,1,1);(2,2,2);(0,1,2)$ khi chia cho $3$ thì chắc chấn ba tấm đó có tổng chia hết cho $3$

có $14$ số chia $3$ dư $1$, $13$ số chia $3$ dư $0$ và $13$ số chia $3$ dư $2$

số cách chọn là $14C_3+13C_3+13C_3+14.13.13=3094$ cách




#594445 $(a^5-a^2+2)(b^5-b^2+2)(c^5-c^2+2)\geq (a+b+c)^3$

Đã gửi bởi Bui Ba Anh on 19-10-2015 - 17:51 trong Bất đẳng thức - Cực trị

Cho a,b,c dương. Chứng minh rằng:
$(a^5-a^2+2)(b^5-b^2+2)(c^5-c^2+2)\geq (a+b+c)^3$

Bài này của Mỹ $2004$, hình như là $+3$ chứ không phải $+2$